Как здесь возникает трение покоя F1,2→F1,2→\vec{F_{1,2}} и F2,1→F2,1→\vec{F_{2,1}}?

введите описание изображения здесь

Как действует статическое трение Ф 1 , 2 и Ф 2 , 1 генерировать здесь? я путаюсь с Ф 1 , 2 и Ф 2 , 1

Мое понимание из видео:- Если мы толкаем блок (2) с силой Ф , Если он движется. Блок (2) находится над блоком (1). Итак, по первому закону инерции Ньютона. Блок (2) имеет тенденцию оставаться в исходном положении. Если есть трение на обеих поверхностях. Когда мы применяем Ф блокировать (1). он будет двигаться правильно. Итак, Блок (2) двигайтесь влево. Итак, возникает трение покоя, препятствующее движению блока в левом направлении. Будь как будет Ф 1 , 2 . Я прав?

Если Блок (1) сдвинут вправо. Нижняя поверхность блока (2) противодействует движению. Итак, это будет Ф 1 , 2 действовать в левом направлении. Верна ли моя логика в обоих случаях?

Ответы (1)

Как здесь возникает трение покоя 𝐹1,2 и 𝐹2,1? Я путаю с 𝐹1,2 и 𝐹2,1

Блок 1 создает статическую силу трения Ф 12 на блок 2 и блок 2 действует равная и противоположная сила трения покоя Ф 21 на блоке 1, пока нет относительного движения. Только блок 1 может заставить блок 2 двигаться вправо благодаря силе трения, которую блок 1 оказывает на блок 2.

Мое понимание из видео:- Если мы толкаем блок (2) с силой 𝐹⃗ , Если он движется. Блок (2) находится над блоком (1). Итак, по первому закону инерции Ньютона. Блок (2) имеет тенденцию оставаться в исходном положении.

Первый закон Ньютона гласит, что покоящееся тело будет оставаться в покое, а тело в движении будет оставаться в движении, ЕСЛИ на него не действует чистая внешняя сила. Блок 2 будет иметь тенденцию оставаться в исходном положении только в том случае, если на него не действуют внешние силы. Но в этом случае на него действует внешняя сила, а именно сила трения блока 1.

Если есть трение на обеих поверхностях. Когда мы применяем 𝐹⃗ к блоку (1). он будет двигаться правильно. Итак, Блок (2) двигайтесь влево. Итак, возникает трение покоя, препятствующее движению блока в левом направлении. Пусть это будет 𝐹1,2→. Я прав?

Если сила Ф действует на блок 1, то блок 2 будет двигаться вместе с блоком 1 вправо до тех пор, пока сила Ф 12 не превышает максимальной силы статического трения мю с м 2 г где мю с - коэффициент статического трения между блоками 1 и 2. В этот момент блок 2 начнет скользить по блоку 1 и испытает кинетическую силу трения, которая обычно меньше силы статического трения. Блок 2 никогда не сдвинется влево относительно системы отсчета опорной поверхности. Он будет двигаться влево относительно системы отсчета блока 1, если будет превышена максимальная сила трения покоя.

Надеюсь это поможет.

пока нет относительного движения. Только блоки 1 могут заставить блок 2 двигаться вправо благодаря силе трения, которую блок 1 оказывает на блок 2. Что вы имеете в виду? не могли бы вы объяснить немного больше?
Почему он не принимает никакой силы реакции на систему. Когда мы применяем Ф ?
@unknownx Что касается первого комментария, посмотрите FBD для блока 2. Единственная внешняя горизонтальная сила, действующая на блок 2, — это сила трения покоя. Ф 12 Направо. Это сила, которая заставит блок ускоряться. Что касается второго комментария, как вы хотите определить «систему»?
Внешняя сила Ф действует на блок 1. правильно? не будет ли блок оказывать равную и противоположную реакцию на человека/поставщика силы?
Ни в одном ФБР они об этом не упоминают. Почему?
@Unknownx Да Ф применяется к блоку 1, но блоки 1 и 2 «слипаются» вместе до тех пор, пока не будет превышена максимальная сила статического трения между 1 и 2, поэтому их можно рассматривать как одно целое. Так что реакция на Ф является реакция блоков 1 и 2. В любом случае, согласно 3-му закону Ньютона, эта реакция также должна быть Ф в противоположном направлении. Почему ни один из FBD не упоминает об этом, вероятно, потому, что это не имеет отношения к определению Ф м а Икс чтобы блок 2 начал проскальзывать. (Вы знаете, как определить Ф м а Икс ?) Меня больше беспокоит, что FBD для блока 1 отсутствует Ф .
Разве они не находят Ф м а Икс путем решения ряда уравнений, полученных из диаграммы свободного тела? Я не понимаю этого утверждения. «Меня больше беспокоит, что в FBD для блока 1 отсутствует F.
@Unknownx Да, они должны быть в состоянии (я не могу воспроизвести видео, чтобы узнать, что они сделали). Я разработал это сам, рассматривая два блока вместе как систему до того, как произойдет проскальзывание блока 2. Суммарная сила, действующая на систему, равна Ф Ф г 1 (единственные две внешние силы, действующие на систему, как определено). Тогда я получил уравнение для ускорения системы, а с у с , что совпадает с ускорением каждого блока в отдельности, пока они остаются вместе.
@Unknownx Тогда сила трения Ф 12 просто м 1 а с у с . Установите его равным максимальной статической силе трения (силе, при которой возможно проскальзывание), мю с 12 м 1 г , и у вас есть выражение для Ф м а Икс по массе и коэффициенту трения. Меня беспокоит FBD для блока 1, так как он не включает Ф . FBD должен показывать все внешние силы, действующие на тело. Может быть, они делают это позже в видео. Надеюсь, это еще поможет.
Большое спасибо.